Last visit was: 23 Apr 2024, 16:19 It is currently 23 Apr 2024, 16:19

Close
GMAT Club Daily Prep
Thank you for using the timer - this advanced tool can estimate your performance and suggest more practice questions. We have subscribed you to Daily Prep Questions via email.

Customized
for You

we will pick new questions that match your level based on your Timer History

Track
Your Progress

every week, we’ll send you an estimated GMAT score based on your performance

Practice
Pays

we will pick new questions that match your level based on your Timer History
Not interested in getting valuable practice questions and articles delivered to your email? No problem, unsubscribe here.
Close
Request Expert Reply
Confirm Cancel
Tags:
Show Tags
Hide Tags
e-GMAT Representative
Joined: 02 Nov 2011
Posts: 4341
Own Kudos [?]: 30776 [108]
Given Kudos: 632
GMAT Date: 08-19-2020
Send PM
e-GMAT Representative
Joined: 02 Nov 2011
Posts: 4341
Own Kudos [?]: 30776 [27]
Given Kudos: 632
GMAT Date: 08-19-2020
Send PM
Director
Director
Joined: 09 Jun 2010
Posts: 530
Own Kudos [?]: 523 [0]
Given Kudos: 916
Send PM
User avatar
Intern
Intern
Joined: 06 Jul 2012
Status:Dedicates 2013 to MBA !!
Posts: 27
Own Kudos [?]: 554 [0]
Given Kudos: 14
Location: United States (MI)
Concentration: Entrepreneurship, General Management
GPA: 3.8
WE:Information Technology (Consulting)
Send PM
Re: How to approach Quantitative Predictive Arguments [#permalink]
Your posts are really clean and informative.
Amazing work !! Thanks !!
Director
Director
Joined: 09 Jun 2010
Posts: 530
Own Kudos [?]: 523 [0]
Given Kudos: 916
Send PM
Re: How to approach Quantitative Predictive Arguments [#permalink]
great, e gmat.

it is clear that cr has some patterns. because if CR has no patterns, gmat can not create questions.

our understanding of patterns is clearly helpful in this game.

I want to name this kind of argument "fuction argument"

evidence: a, b increase/decrease
conclusion: Z =f (a,b,c) increase/decrease
prethink assumption: c (other variables) dose not change

once we can prethink an assumption, everything would be easy.

pls, give us pdf of this article. Thank you
Director
Director
Joined: 09 Jun 2010
Posts: 530
Own Kudos [?]: 523 [0]
Given Kudos: 916
Send PM
Re: How to approach Quantitative Predictive Arguments [#permalink]
this article is applicable and effective when we solve the question because it is simple to apply and remember. we nearly do not need to remember any thing. the logic is easy to remember.

whenever we see "decrease, increase, equal, go up, go down" in the argument, it is possible that this argument appear.

the time is reduced dramatically
e-GMAT Representative
Joined: 02 Nov 2011
Posts: 4341
Own Kudos [?]: 30776 [8]
Given Kudos: 632
GMAT Date: 08-19-2020
Send PM
Re: How to approach Quantitative Predictive Arguments [#permalink]
8
Kudos
Expert Reply
CharuKapoor wrote:
Your posts are really clean and informative.
Amazing work !! Thanks !!


Thanks Charu for the appreciation! :)

Such appreciation from forum members keeps us motivated.

Also, feel free to give Kudos whenever you like our posts. We don't mind getting Kudos :wink:

Thanks,
Chiranjeev
e-GMAT Representative
Joined: 02 Nov 2011
Posts: 4341
Own Kudos [?]: 30776 [4]
Given Kudos: 632
GMAT Date: 08-19-2020
Send PM
Re: How to approach Quantitative Predictive Arguments [#permalink]
4
Kudos
Expert Reply
The answer choices for the 10 questions posted are:

A
A
A
A
B
A
A
A
C
B

Thanks,
Chiranjeev
avatar
Intern
Intern
Joined: 15 Aug 2012
Posts: 38
Own Kudos [?]: 85 [2]
Given Kudos: 2
Send PM
Re: How to approach Quantitative Predictive Arguments [#permalink]
2
Kudos
egmat wrote:

A FRAMEWORK TO PRETHINK IN QUANTITATIVE PREDICTIVE ARGUMENTS



In the past the country of Malvernia has relied heavily on imported oil. Malvernia recently implemented a program to convert heating systems from oil to natural gas. Malvernia currently produces more natural gas each year than it uses, and oil production in Malvernian oil fields is increasing at a steady pace. If these trends in fuel production and usage continue, therefore, Malvernian reliance on foreign sources for fuel is likely to decline soon.

Which of the following would it be most useful to establish in evaluating the argument?

(A) When, if ever, will production of oil in Malvernia outstrip production of natural gas?

(B) Is Malvernia among the countries that rely most on imported oil?

(C) What proportion of Malvernia's total energy needs is met by hydroelectric, solar, and nuclear power?

(D) Is the amount of oil used each year in Malvernia for generating electricity and fuel for transportation increasing?

(E) Have any existing oil-burning heating systems in Malvernia already been converted to natural-gas-burning heating systems?

The question is talking about import of fuel to meet the deficit in demand and supply. In equation form, it is:

    Import = Demand – Supply


    Import of Natural Gas and Oil = (Demand – Supply)NG + (Demand – Supply)oil


Now, we know that the first parenthesized expression is positive and in the second expression, we are only told about supply of oil, not the demand, so the answer choice must be talking about demand of oil.



Hi Chiranjeev,

Excellent article !!

I just had 1 clarification for the second question above.

The conclusion above is Malvernian reliance on foreign sources for fuel is likely to decline soon. and the equation set up is

    Import of Natural Gas and Oil = (Demand – Supply)NG + (Demand – Supply)oil


so what we are saying is that the left hand side of the expression has to reduce. this can happen in 2 possible ways
1. both the expressions on the right reduce
2. 1 expression reduces and the other either remains the same or does not increase enough to compensate for the decrease.

Now per the premise, Malvernian produces more natural gas than it uses. it means that demand is less than supply. so your first expression for natural gas will become negative and not positive(your article states positive). and the premise also gives only the supply information of oil, so to confirm the conclusion we will talk about the demand of oil, whether the demand is stable or increasing.

Is this correct ?
e-GMAT Representative
Joined: 02 Nov 2011
Posts: 4341
Own Kudos [?]: 30776 [1]
Given Kudos: 632
GMAT Date: 08-19-2020
Send PM
Re: How to approach Quantitative Predictive Arguments [#permalink]
1
Kudos
Expert Reply
Dinesh2Apr wrote:
egmat wrote:

A FRAMEWORK TO PRETHINK IN QUANTITATIVE PREDICTIVE ARGUMENTS



In the past the country of Malvernia has relied heavily on imported oil. Malvernia recently implemented a program to convert heating systems from oil to natural gas. Malvernia currently produces more natural gas each year than it uses, and oil production in Malvernian oil fields is increasing at a steady pace. If these trends in fuel production and usage continue, therefore, Malvernian reliance on foreign sources for fuel is likely to decline soon.

Which of the following would it be most useful to establish in evaluating the argument?

(A) When, if ever, will production of oil in Malvernia outstrip production of natural gas?

(B) Is Malvernia among the countries that rely most on imported oil?

(C) What proportion of Malvernia's total energy needs is met by hydroelectric, solar, and nuclear power?

(D) Is the amount of oil used each year in Malvernia for generating electricity and fuel for transportation increasing?

(E) Have any existing oil-burning heating systems in Malvernia already been converted to natural-gas-burning heating systems?

The question is talking about import of fuel to meet the deficit in demand and supply. In equation form, it is:

    Import = Demand – Supply


    Import of Natural Gas and Oil = (Demand – Supply)NG + (Demand – Supply)oil


Now, we know that the first parenthesized expression is positive and in the second expression, we are only told about supply of oil, not the demand, so the answer choice must be talking about demand of oil.



Hi Chiranjeev,

Excellent article !!

I just had 1 clarification for the second question above.

The conclusion above is Malvernian reliance on foreign sources for fuel is likely to decline soon. and the equation set up is

    Import of Natural Gas and Oil = (Demand – Supply)NG + (Demand – Supply)oil


so what we are saying is that the left hand side of the expression has to reduce. this can happen in 2 possible ways
1. both the expressions on the right reduce
2. 1 expression reduces and the other either remains the same or does not increase enough to compensate for the decrease.

Now per the premise, Malvernian produces more natural gas than it uses. it means that demand is less than supply. so your first expression for natural gas will become negative and not positive(your article states positive). and the premise also gives only the supply information of oil, so to confirm the conclusion we will talk about the demand of oil, whether the demand is stable or increasing.

Is this correct ?


Hi Dinesh,

Very interesting doubt! I am glad that you went through the article so thoroughly.

First of all, thanks for pointing out the error. The expression should be negative. The same has been corrected in the article. However, it doesn't effect the solution since we are concerned with decline (which means change), not the current values.

You are right that reduction can happen in two ways. However, let's look at the question - it says that the current trends will continue. What are these trends? There are two trends:
1. Supply of NG > Demand of NG
2. Oil Production (Supply of oil) increasing

Now, the reduction in import would not be there only if the demand is increasing (such that the increase in demand is more than increase in supply of oil). Pay attention here, we are talking in terms of increase (change) in both the demand and the supply - not their current absolute values.

So, in the given situation, to test the validity of the conclusion, I need to know whether the demand for oil is increasing or not. If the demand is stable or decreasing, the conclusion will always hold. Only when demand is increasing, we become doubtful of the conclusion.

Hope this helps :)

Let me know if I missed something.

Thanks,
Chiranjeev
User avatar
Manager
Manager
Joined: 09 Nov 2012
Status:How easy it is?
Posts: 74
Own Kudos [?]: 408 [0]
Given Kudos: 174
Location: India
Concentration: Operations, General Management
GMAT 1: 650 Q50 V27
GMAT 2: 710 Q49 V37
GPA: 3.5
WE:Operations (Other)
Send PM
Re: How to approach Quantitative Predictive Arguments [#permalink]
egmat wrote:
Here are the 10 OG questions as promised. Use the methods described above to solve these questions in less than 1 minute

1. The cotton farms of Country Q became so productive that the market could not absorb all that they produced. Consequently, cotton prices fell. The government tried to boost cotton prices by offering farmers who took 25 percent of their cotton acreage out of production direct support payments up to a specified maximum per farm.
The government’s program, if successful, will not be a net burden on the budget. Which of the following, if true, is the best basis for an explanation of how this could be so?
A. Depressed cotton prices meant operating losses for cotton farms, and the government lost revenue from taxes on farm profits.
B. Cotton production in several counties other than Q declined slightly the year that the support-payment program went into effect in Q.
C. The first year that the support-payment program was in effect, cotton acreage in Q was 5% below its level in the base year for the program.
D. The specified maximum per farm meant that for very large cotton farms the support payments were less per acre for those acres that were withdrawn from production than they were for smaller farms.
E. Farmers who wished to qualify for support payments could not use the cotton acreage that was withdrawn from production to grow any other crop.

2. In Gandania, where the government has a monopoly on tobacco sales, the incidence of smoking-related health problems has risen steadily for the last twenty years. The health secretary recently proposed a series of laws aimed at curtailing tobacco use in Gandania. Profits from tobacco sales, however, account for ten percent of Gandania’s annual revenues. Therefore, Gandania cannot afford to institute the proposed laws.

Which of the following, if true, most seriously weakens the argument?
A. All health care in Gandania is government-funded.
B. Implementing the proposed laws is not likely to cause a significant increase in the amount of tobacco Gandania exports.
C. The percentage of revenue Gandania receives from tobacco sales has remained steady in recent years.
D. Profits from tobacco sales far surpass any other single source of revenue for the Gandanian government.
E. No government official in Gandania has ever previously proposed laws aimed at curtailing tobacco use.

3. Printwell’s Ink Jet Division manufactures ink-jet printers and the ink cartridges they use. Sales of its ink-jet printers have increased. Monthly revenues from those sales, however, have not increased, because competition has forced Printwell to cut the prices of its printers. Unfortunately, Printwell has been unable to bring down the cost of manufacturing a printer. Thus, despite the increase in printer sales, the Ink Jet Division must be providing the company with much smaller than it used to.
Which of the following, if true, most seriously weakens the argument?
A. Ink-jet printers in regular use frequently need new ink cartridges, and Printwell’s printers only accept Printwell’s ink cartridges.
B. Unlike some competing companies, Printwell sells all of its printers through retailers, and these retailers’ costs account for a sizable proportion of the printers’ ultimate retail price.
C. Some printer manufacturers have been forced to reduce the sale price of their ink-jet printers even more than Printwell has.
D. In the past year, no competing manufacturer of ink-jet printers has had as great an increase in unit sales of printers as Printwell has.
E. In the past year, sales of Printwell’s ink-jet printers have increased more than sales of any other type of printer made by Printwell.

4. In Teruvia, the quantity of rice produced per year is currently just large enough to satisfy domestic demand. Teruvia’s total rice acreage will not be expanded in the foreseeable future, nor will rice yields per acre increase appreciably. Teruvia’s population, however, will be increasing significantly for years to come. Clearly, therefore, Teruvia will soon have to begin importing rice.
Which of the following is an assumption on which the argument depends?
A. No pronounced trend of decreasing per capita demand for rice is imminent in Teruvia.
B. Not all of the acreage in Teruvia currently planted with rice is well suited to the cultivation of rice.
C. None of the strains of rice grown in Teruvia are exceptionally high-yielding.
D. There are no populated regions in Teruvia in which the population will not increase.
E. There are no major crops other than rice for which domestic production and domestic demand are currently in balance in Teruvia.

5. For similar cars and comparable drivers, automobile insurance for collision damage has always cost more in Greatport than in Fairmont. Police studies, however, show that cars owned by Greatport residents are, on average, slightly less likely to be involved in a collision than cars in Fairmont. Clearly, therefore, insurance companies are making a greater profit on collision damage insurance in Greatport than in Fairmont.
In evaluating the argument, it would be most useful to compare
A. the level of traffic congestion in Greatport with the level of traffic congestion in Fairmont
B. the cost of repairing collision damage in Greatport with the cost of repairing collision damage in Fairmont
C. the rates Greatport residents pay for other forms of insurance with the rates paid for similar insurance by residents of Fairmont
D. the condition of Greatport's roads and streets with the condition of Fairmont's roads and streets
E. the cost of collision-damage insurance in Greatport and Fairmont with that in other cities

6. Contrary to earlier predictions, demand for sugarcane has not increased in recent years. Yet, even though prices and production amounts have also been stable during the last three years, sugarcane growers last year increased their profits by more than 10% over their previous year's level.
Any of the following statements, if true about last year, helps to explain the rise in profits EXCEPT:
A. Many countries that are large consumers of sugarcane increased their production of sugarcane-based ethanol. yet their overall consumption decreased.
B. Sugarcane growers have saved money on wages by switching from paying laborers an hourly wage to paying them by the amount harvested.
C. The price of the oil, the major energy source used by sugarcane growers in harvesting their crops, dropped by more than 20%.
D. Many small sugarcane growers joined together to form an association of sugarcane producers and began to buy supplies at low group rates.
E. Rainfall in sugarcane-growing regions was higher than it had been during the previous year, allowing the growers to save money on expensive artificial irrigation.

7. Which of the following most logically completes the argument below?
Davison River farmers are currently deciding between planting winter wheat this fall or spring wheat next spring. Winter wheat and spring wheat are usually about equally profitable. Because of new government restrictions on the use of Davison River water for irrigation, per acre yields for winter wheat, though not for spring wheat, would be much lower than average. Therefore, planting spring wheat will be more profitable than planting winter wheat, since_______.
A. the smaller-than-average size of a winter wheat harvest this year would not be compensated for by higher winter wheat prices
B. new crops of spring wheat must be planted earlier than the time at which standing crops of winter wheat are ready to be harvested
C. the spring wheat that farmers in the Davison River region plant is well adapted to the soil of the region
D. spring wheat has uses that are different from those of winter wheat
E. planting spring wheat is more profitable than planting certain other crops, such as rye

8. Insurance Company X is considering issuing a new policy to cover services required by elderly people who suffer from diseases that afflict the elderly. Premiums for the policy must be low enough to attract customers. Therefore, Company X is concerned that the income from the policies would not be sufficient to pay for the claims that would be made.
Which of the following strategies would be most likely to minimize Company X's losses on the policies?
A. Attracting middle-aged customers unlikely to submit claims for benefits for many years
B. Insuring only those individuals who did not suffer any serious diseases as children
C. Including a greater number of services in the policy than are included in other policies of lower cost
D. Insuring only those individuals who were rejected by other companies for similar policies
E. Insuring only those individuals who are wealthy enough to pay for the medical services

9. In the United States, of the people who moved from one state to another when they retired, the percentage who retired to Florida has decreased by three percentage points over the past ten years. Since many local businesses in Florida cater to retirees, these declines are likely to have a noticeably negative economic effect on these businesses and therefore on the economy of Florida.
Which of the following, if true, most seriously weakens the argument given?
A. People who moved from one state to another when they retired moved a greater distance, on average, last year than such people did ten years ago.
B. People were more likely to retire to North Carolina from another state last year than people were ten years ago.
C. The number of people who moved from one state to another when they retired has increased significantly over the past ten years.
D. The number of people who left Florida when they retired to live in another state was greater last year than it was ten years ago.
E. Florida attracts more people who move from one state to another when they retire than does any other state.

10. A discount retailer of basic household necessities employs thousands of people and pays most of them at the minimum wage rate. Yet following a federally mandated increase of the minimum wage rate that increased the retailer’s operating costs considerably, the retailer’s profits increased markedly.
Which of the following, if true, most helps to resolve the apparent paradox?
A. Over half of the retailer’s operating costs consist of payroll expenditures; yet only a small percentage of those expenditures go to pay management salaries.
B. The retailer’s customer base is made up primarily of people who earn, or who depend on the earnings of others who earn, the minimum wage.
C. The retailer’s operating costs, other than wages, increased substantially after the increase in the minimum wage rate went into effect.
D. When the increase in the minimum wage rate went into effect, the retailer also raised the age rate for employees who had been earning just above minimum wage.
E. The majority of the retailer’s employees’ work as cashiers, and most cashiers are paid the minimum wage.



Kindly post the explanations with equations!!

Thanks
avatar
Intern
Intern
Joined: 05 Aug 2012
Posts: 12
Own Kudos [?]: 6 [0]
Given Kudos: 2
Send PM
Re: A FRAMEWORK TO PRETHINK IN QUANTITATIVE PREDICTIVE ARGUMENTS [#permalink]
Yeah... Please post the answers with explanations... Although I could make out the answer, for some problems I couldnt make out the equation...or rather I wanted to be sure that the equation that I made is correct or not! :)
avatar
Manager
Manager
Joined: 03 Jul 2013
Status:Got Bling! Joined Phd Finance at IIML
Affiliations: IIMB, advantages.us, IIML
Posts: 62
Own Kudos [?]: 23 [0]
Given Kudos: 39
Location: India
Concentration: Finance, Economics
Schools: iim-bangalore - Class of 1994
GMAT 1: 750 Q59 V43
GPA: 3.12
WE:Research (Investment Banking)
Send PM
Re: A FRAMEWORK TO PRETHINK IN QUANTITATIVE PREDICTIVE ARGUMENTS [#permalink]
Quote:
1. The cotton farms of Country Q became so productive that the market could not absorb all that they produced. Consequently, cotton prices fell. The government tried to boost cotton prices by offering farmers who took 25 percent of their cotton acreage out of production direct support payments up to a specified maximum per farm.
The government’s program, if successful, will not be a net burden on the budget. Which of the following, if true, is the best basis for an explanation of how this could be so?
A. Depressed cotton prices meant operating losses for cotton farms, and the government lost revenue from taxes on farm profits.
B. Cotton production in several counties other than Q declined slightly the year that the support-payment program went into effect in Q.
C. The first year that the support-payment program was in effect, cotton acreage in Q was 5% below its level in the base year for the program.
D. The specified maximum per farm meant that for very large cotton farms the support payments were less per acre for those acres that were withdrawn from production than they were for smaller farms.
E. Farmers who wished to qualify for support payments could not use the cotton acreage that was withdrawn from production to grow any other crop.


A and B both support the case, however B requires other conditions ( international prices) to move to justify spend on support

go with A

1. A
avatar
Manager
Manager
Joined: 03 Jul 2013
Status:Got Bling! Joined Phd Finance at IIML
Affiliations: IIMB, advantages.us, IIML
Posts: 62
Own Kudos [?]: 23 [0]
Given Kudos: 39
Location: India
Concentration: Finance, Economics
Schools: iim-bangalore - Class of 1994
GMAT 1: 750 Q59 V43
GPA: 3.12
WE:Research (Investment Banking)
Send PM
Re: A FRAMEWORK TO PRETHINK IN QUANTITATIVE PREDICTIVE ARGUMENTS [#permalink]
Quote:
2. In Gandania, where the government has a monopoly on tobacco sales, the incidence of smoking-related health problems has risen steadily for the last twenty years. The health secretary recently proposed a series of laws aimed at curtailing tobacco use in Gandania. Profits from tobacco sales, however, account for ten percent of Gandania’s annual revenues. Therefore, Gandania cannot afford to institute the proposed laws.

Which of the following, if true, most seriously weakens the argument?
A. All health care in Gandania is government-funded.
B. Implementing the proposed laws is not likely to cause a significant increase in the amount of tobacco Gandania exports.
C. The percentage of revenue Gandania receives from tobacco sales has remained steady in recent years.
D. Profits from tobacco sales far surpass any other single source of revenue for the Gandanian government.
E. No government official in Gandania has ever previously proposed laws aimed at curtailing tobacco use.


A. Directly weakens the argument but whether impact is significant comparison cannot be made ( motive in other choices)
B. supports argument if data is available
C. does not weaken argument
D. strengthens argument
E. could have weakened argument if data was available on ineffectiveness of such curtailment on sales

Go with the best answer A
2. A
avatar
Manager
Manager
Joined: 03 Jul 2013
Status:Got Bling! Joined Phd Finance at IIML
Affiliations: IIMB, advantages.us, IIML
Posts: 62
Own Kudos [?]: 23 [0]
Given Kudos: 39
Location: India
Concentration: Finance, Economics
Schools: iim-bangalore - Class of 1994
GMAT 1: 750 Q59 V43
GPA: 3.12
WE:Research (Investment Banking)
Send PM
Re: A FRAMEWORK TO PRETHINK IN QUANTITATIVE PREDICTIVE ARGUMENTS [#permalink]
Quote:
3. Printwell’s Ink Jet Division manufactures ink-jet printers and the ink cartridges they use. Sales of its ink-jet printers have increased. Monthly revenues from those sales, however, have not increased, because competition has forced Printwell to cut the prices of its printers. Unfortunately, Printwell has been unable to bring down the cost of manufacturing a printer. Thus, despite the increase in printer sales, the Ink Jet Division must be providing the company with much smaller than it used to.
Which of the following, if true, most seriously weakens the argument?
A. Ink-jet printers in regular use frequently need new ink cartridges, and Printwell’s printers only accept Printwell’s ink cartridges.
B. Unlike some competing companies, Printwell sells all of its printers through retailers, and these retailers’ costs account for a sizable proportion of the printers’ ultimate retail price.
C. Some printer manufacturers have been forced to reduce the sale price of their ink-jet printers even more than Printwell has.
D. In the past year, no competing manufacturer of ink-jet printers has had as great an increase in unit sales of printers as Printwell has.
E. In the past year, sales of Printwell’s ink-jet printers have increased more than sales of any other type of printer made by Printwell.


D and E do not support the argument of profits in question directly from inkjet printers
C again sdoes not reflect on PW inkjet profits
B whether retaier costs have been tweaked can be tweaked is not available
A remains valid and a profitable venture

The answer is

3. A
avatar
Manager
Manager
Joined: 03 Jul 2013
Status:Got Bling! Joined Phd Finance at IIML
Affiliations: IIMB, advantages.us, IIML
Posts: 62
Own Kudos [?]: 23 [0]
Given Kudos: 39
Location: India
Concentration: Finance, Economics
Schools: iim-bangalore - Class of 1994
GMAT 1: 750 Q59 V43
GPA: 3.12
WE:Research (Investment Banking)
Send PM
Re: A FRAMEWORK TO PRETHINK IN QUANTITATIVE PREDICTIVE ARGUMENTS [#permalink]
Quote:
4. In Teruvia, the quantity of rice produced per year is currently just large enough to satisfy domestic demand. Teruvia’s total rice acreage will not be expanded in the foreseeable future, nor will rice yields per acre increase appreciably. Teruvia’s population, however, will be increasing significantly for years to come. Clearly, therefore, Teruvia will soon have to begin importing rice.
Which of the following is an assumption on which the argument depends?
A. No pronounced trend of decreasing per capita demand for rice is imminent in Teruvia.
B. Not all of the acreage in Teruvia currently planted with rice is well suited to the cultivation of rice.
C. None of the strains of rice grown in Teruvia are exceptionally high-yielding.
D. There are no populated regions in Teruvia in which the population will not increase.
E. There are no major crops other than rice for which domestic production and domestic demand are currently in balance in Teruvia.


4. A Directly related to question's two arguments, others impact trend positively or negatively and cannot/need not be evaluated as acreage and yield and aggregate demand already given
avatar
Manager
Manager
Joined: 03 Jul 2013
Status:Got Bling! Joined Phd Finance at IIML
Affiliations: IIMB, advantages.us, IIML
Posts: 62
Own Kudos [?]: 23 [1]
Given Kudos: 39
Location: India
Concentration: Finance, Economics
Schools: iim-bangalore - Class of 1994
GMAT 1: 750 Q59 V43
GPA: 3.12
WE:Research (Investment Banking)
Send PM
Re: A FRAMEWORK TO PRETHINK IN QUANTITATIVE PREDICTIVE ARGUMENTS [#permalink]
1
Kudos
Quote:
5. For similar cars and comparable drivers, automobile insurance for collision damage has always cost more in Greatport than in Fairmont. Police studies, however, show that cars owned by Greatport residents are, on average, slightly less likely to be involved in a collision than cars in Fairmont. Clearly, therefore, insurance companies are making a greater profit on collision damage insurance in Greatport than in Fairmont.
In evaluating the argument, it would be most useful to compare
A. the level of traffic congestion in Greatport with the level of traffic congestion in Fairmont
B. the cost of repairing collision damage in Greatport with the cost of repairing collision damage in Fairmont
C. the rates Greatport residents pay for other forms of insurance with the rates paid for similar insurance by residents of Fairmont
D. the condition of Greatport's roads and streets with the condition of Fairmont's roads and streets
E. the cost of collision-damage insurance in Greatport and Fairmont with that in other cities


A. can work into data if relation was provided. not objective to this solution.
B. Probably the Right answer, decdes premium values
C. irrelevant
D. can work into data if relation was provided. not objective to this solution.
E. irrelevant

The answer is B

5. B
avatar
Manager
Manager
Joined: 03 Jul 2013
Status:Got Bling! Joined Phd Finance at IIML
Affiliations: IIMB, advantages.us, IIML
Posts: 62
Own Kudos [?]: 23 [0]
Given Kudos: 39
Location: India
Concentration: Finance, Economics
Schools: iim-bangalore - Class of 1994
GMAT 1: 750 Q59 V43
GPA: 3.12
WE:Research (Investment Banking)
Send PM
Re: A FRAMEWORK TO PRETHINK IN QUANTITATIVE PREDICTIVE ARGUMENTS [#permalink]
Quote:
6. Contrary to earlier predictions, demand for sugarcane has not increased in recent years. Yet, even though prices and production amounts have also been stable during the last three years, sugarcane growers last year increased their profits by more than 10% over their previous year's level.
Any of the following statements, if true about last year, helps to explain the rise in profits EXCEPT:
A. Many countries that are large consumers of sugarcane increased their production of sugarcane-based ethanol. yet their overall consumption decreased.
B. Sugarcane growers have saved money on wages by switching from paying laborers an hourly wage to paying them by the amount harvested.
C. The price of the oil, the major energy source used by sugarcane growers in harvesting their crops, dropped by more than 20%.
D. Many small sugarcane growers joined together to form an association of sugarcane producers and began to buy supplies at low group rates.
E. Rainfall in sugarcane-growing regions was higher than it had been during the previous year, allowing the growers to save money on expensive artificial irrigation.


Half the costs need to decrease by 20% to make that increase in profit. or half the sales need to increase realisation by 20% etc

A already looks the answer for it actually does not increase profits and may decrease it as required in "EXCEPT"
B. reduces costs so could explain
C. definitely explains
D. costs decreased , could explain
E. costs of irrigation steeply down, explains

the answer is A

6. A
avatar
Manager
Manager
Joined: 03 Jul 2013
Status:Got Bling! Joined Phd Finance at IIML
Affiliations: IIMB, advantages.us, IIML
Posts: 62
Own Kudos [?]: 23 [0]
Given Kudos: 39
Location: India
Concentration: Finance, Economics
Schools: iim-bangalore - Class of 1994
GMAT 1: 750 Q59 V43
GPA: 3.12
WE:Research (Investment Banking)
Send PM
Re: A FRAMEWORK TO PRETHINK IN QUANTITATIVE PREDICTIVE ARGUMENTS [#permalink]
Quote:
7. Which of the following most logically completes the argument below?
Davison River farmers are currently deciding between planting winter wheat this fall or spring wheat next spring. Winter wheat and spring wheat are usually about equally profitable. Because of new government restrictions on the use of Davison River water for irrigation, per acre yields for winter wheat, though not for spring wheat, would be much lower than average. Therefore, planting spring wheat will be more profitable than planting winter wheat, since_______.
A. the smaller-than-average size of a winter wheat harvest this year would not be compensated for by higher winter wheat prices
B. new crops of spring wheat must be planted earlier than the time at which standing crops of winter wheat are ready to be harvested
C. the spring wheat that farmers in the Davison River region plant is well adapted to the soil of the region
D. spring wheat has uses that are different from those of winter wheat
E. planting spring wheat is more profitable than planting certain other crops, such as rye


My answer is B as the winter wheat could otherwise be followed by winter wheat making the profits from spring enough to compensate for the year's lower winter what numbers
A very material but not the best choice as it does not support spring wheat or overall profit with explanationf or B above
B. as above
C. does not matter in data
D. is irrelevant
E. is irrelevant
avatar
Manager
Manager
Joined: 03 Jul 2013
Status:Got Bling! Joined Phd Finance at IIML
Affiliations: IIMB, advantages.us, IIML
Posts: 62
Own Kudos [?]: 23 [0]
Given Kudos: 39
Location: India
Concentration: Finance, Economics
Schools: iim-bangalore - Class of 1994
GMAT 1: 750 Q59 V43
GPA: 3.12
WE:Research (Investment Banking)
Send PM
Re: A FRAMEWORK TO PRETHINK IN QUANTITATIVE PREDICTIVE ARGUMENTS [#permalink]
Quote:
8. Insurance Company X is considering issuing a new policy to cover services required by elderly people who suffer from diseases that afflict the elderly. Premiums for the policy must be low enough to attract customers. Therefore, Company X is concerned that the income from the policies would not be sufficient to pay for the claims that would be made.
Which of the following strategies would be most likely to minimize Company X's losses on the policies?
A. Attracting middle-aged customers unlikely to submit claims for benefits for many years
B. Insuring only those individuals who did not suffer any serious diseases as children
C. Including a greater number of services in the policy than are included in other policies of lower cost
D. Insuring only those individuals who were rejected by other companies for similar policies
E. Insuring only those individuals who are wealthy enough to pay for the medical services


A profitable but if the segment will apply to product for elders or can be included
B allowable but may not be correlated to lower incidence of elderly disease to bring down premiums , or ensure profit
C Can attract customers as included in problem statement
D does not cover problem statement as they would pay high premiums
E does not cover problem statement as they would pay high premiums

A seems to be the better choice

8. A
GMAT Club Bot
Re: A FRAMEWORK TO PRETHINK IN QUANTITATIVE PREDICTIVE ARGUMENTS [#permalink]
 1   2   3   
Moderators:
GMAT Club Verbal Expert
6917 posts
GMAT Club Verbal Expert
238 posts
GRE Forum Moderator
13957 posts

Powered by phpBB © phpBB Group | Emoji artwork provided by EmojiOne